首页 | 本学科首页   官方微博 | 高级检索  
相似文献
 共查询到20条相似文献,搜索用时 35 毫秒
1.
In this paper, we prove that if {a,b,c,d} is a set of four non-zero polynomials with integer coefficients, not all constant, such that the product of any two of its distinct elements plus 1 is a square of a polynomial with integer coefficients, then
(a+b−c−d)2=4(ab+1)(cd+1).  相似文献   

2.
Let m?−1 be an integer. We give a correspondence between integer solutions to the parametric family of cubic Thue equations
X3mX2Y−(m+3)XY2Y3=λ  相似文献   

3.
Given a number field K and a subgroup GK of the multiplicative group of K, Silverman defined the G-height H(θ;G) of an algebraic number θ as
  相似文献   

4.
A new simple method for approximating certain algebraic numbers is developed. By applying this method, an effective upper bound is derived for the integral solutions of the quartic Thue equation with two parameters $tx^4 - 4sx^3 y - 6tx^2 y^2 + 4sxy^3 + ty^4 = N$ , where s > 32t 3. As an application, Ljunggren’s equation is solved in an elementary way.  相似文献   

5.
In 2001, Borwein, Choi, and Yazdani looked at an extremal property of a class of polynomial with ±1 coefficients. Their key result was:
Theorem. (See Borwein, Choi, Yazdani, 2001.) Letf(z)=±z±z2±?±zN−1, and ζ a primitive Nth root of unity. If N is an odd positive integer then
  相似文献   

6.
Let r be an integer not less than 2. Suppose that we have a (not necessarily homogeneous) Markov chain with state space {0,1,…,r−1} given by the sequence of r×r transition matrices
  相似文献   

7.
Let G be an Abelian group with a metric d and E a normed space. For any f:G→E we define the quadratic difference of the function f by the formula
Qf(x,y):=2f(x)+2f(y)−f(x+y)−f(x−y)  相似文献   

8.
Let (Cn)n?0 be the Lucas sequence Cn+2=aCn+1+bCn for all n?0, where C0=0 and C1=1. For 1?k?m−1 let
  相似文献   

9.
Let f(x) be a real valued polynomial in x of degree k?4 with leading coefficient α. In this paper, we prove a non-trivial upper bound for the quantity
  相似文献   

10.
Let λ(n) be the nth normalized Fourier coefficient of a holomorphic Hecke eigencuspform f(z) of even integral weight k for the full modular group. In this paper we are able to prove the following results.
(i)
For any ε>0, we have
  相似文献   

11.
Let Pr denote an almost prime with at most r prime factors, counted according to multiplicity. In the present paper, it is proved that for any sufficiently large even integer n, the equation
$$n = {x^3} + p_1^3 + p_2^3 + p_3^3 + p_4^3 + p_5^3 + p_6^4 + p_7^4$$
has solutions in primes pi with x being a P6. This result constitutes a refinement upon that of Hooley C.
  相似文献   

12.
For a class of strictly increasing real valued functions f(n) we obtain an upper bound for the number of solutions of the equation
  相似文献   

13.
This paper contains new estimates for the distance between adjacent zeros of solutions of the first order delay differential equation
x(t)+p(t)x(tτ)=0  相似文献   

14.
Suppose that λ1, λ2, λ3, λ4 are nonzero real numbers, not all negative, δ > 0, V is a well-spaced set, and the ratio λ12 is algebraic and irrational. Denote by E(V,N, δ) the number of vV with vN such that the inequality
$$\left| {{\lambda _1}p_1^2 + {\lambda _2}p_2^3 + {\lambda _3}p_3^4 + {\lambda _4}p_4^5 - \upsilon } \right| < {\upsilon ^{ - \delta }}$$
has no solution in primes p1, p2, p3, p4. We show that
$$E\left( {\upsilon ,N,\delta } \right) \ll {N^{1 + 2\delta - 1/72 + \varepsilon }}$$
for any ? > 0.
  相似文献   

15.
Let q?2 be an integer, χ be any non-principal character mod q, and H=H(q)?q. In this paper the authors prove some estimates for character sums of the form
  相似文献   

16.
We count the number S(x) of quadruples \( {\left( {x_{1} ,x_{2} ,x_{3} ,x_{4} } \right)} \in \mathbb{Z}^{4} \) for which
$ p = x^{2}_{1} + x^{2}_{2} + x^{2}_{3} + x^{2}_{4} \leqslant x $
is a prime number and satisfying the determinant condition: x 1 x 4???x 2 x 3?=?1. By means of the sieve, one shows easily the upper bound S(x)???x/log x. Under a hypothesis about prime numbers, which is stronger than the Bombieri–Vinogradov theorem but is weaker than the Elliott–Halberstam conjecture, we prove that this order is correct, that is S(x)???x/log x.
  相似文献   

17.
Let (|q|<1). For kN it is shown that there exist k rational numbers A(k,0),…,A(k,k−1) such that
  相似文献   

18.
By some extremely simple arguments, we point out the following:
(i)
If n is the least positive kth power non-residue modulo a positive integer m, then the greatest number of consecutive kth power residues mod m is smaller than m/n.
(ii)
Let OK be the ring of algebraic integers in a quadratic field with d∈{−1,−2,−3,−7,−11}. Then, for any irreducible πOK and positive integer k not relatively prime to , there exists a kth power non-residue ωOK modulo π such that .
  相似文献   

19.
20.
Let k,m,n?2 be integers. Let A be a subset of {0,1,…,n} with 0∈A and the greatest common divisor of all elements of A is 1. Suppose that
  相似文献   

设为首页 | 免责声明 | 关于勤云 | 加入收藏

Copyright©北京勤云科技发展有限公司  京ICP备09084417号